SBA 300 Paper 6 Flashcards

1
Q
  1. A 44-year-old woman is on the intensive care unit having had a grade 3
    subarachnoid haemorrhage secondary to an anterior communicating artery
    aneurysm one day ago. She is currently stable neurologically. Her past medical history comprises of hypercholesterolaemia, hypertension and smoking. She has a drug history of simvastatin and lisinopril.
    Which of the following would most likely prevent the development of delayed cerebral ischaemia in this patient?
    A ‘Triple H therapy’
    B Magnesium administration
    C Statin administration
    D Nimodipine administration
    E Antiplatelet therap
A
  1. D Nimodipine administration
    ○ Delayed cerebral ischaemia describes neurological deterioration that occurs secondary to ischaemia alone (i.e. not hydrocephalus or seizure activity) and persists for greater than 1 hour.
    ○ It develops in more than 60% of subarachnoid haemorrhage
    (SAH) patients and confers a less favourable outcome.
    ° Patients are at greatest risk of ischaemia from day 3 to day 10 post-SAH. ° Their risk is also augmented by a poor grade of SAH (Table 6.2), a large volume haemorrhage within the subarachnoid space or extending to the ventricles and a smoking history.
    ○ Delayed ischaemia is frequently labelled as intracranial vasospasm, but until confirmed by investigation the two terms should be separately defined. They are treated in an identical fashion.
    ○ The use of triple H therapy (hypertension, hypervolaemia and haemodilution) is now controversial.
    °Those who advocate it do so in order to improve cerebral blood flow by increasing cerebral perfusion pressure (CPP), volume status and blood rheology.
    ° Targets for each are CPP >70mmHg, CVP 12–15mmHg and haematocrit
    0.3 respectively.
    ° More recent studies have failed to show conclusive benefits from any element but it is widely accepted that hypovolaemia and hypotension are deleterious.
    ° The patient’s premorbid blood pressure must also be acknowledged when calculating a suitable target.
    ○ Hypermagnesaemia has been promoted as reducing the risk of cerebral ischaemia as magnesium is a vasodilator and thought to have a role in neuroprotection.
    ° The 2011 IMASH trial (Intravenous Magnesium Sulphate in Aneurysmal Subarachnoid Haemorrhage) failed to show any benefit but it is felt that further studies are needed to ascertain the optimum level required to improve outcome.
    ° A magnesium level below the normal range should definitely be corrected.
    ○ In addition to treating hypercholesterolaemia, statins have been found to modulate the cytokine response.
    ° They also reduce the quantity of reactive oxygen molecules produced in brain injury.
    ° Overall, the subsequent inflammatory response is minimised and they have therefore been suggested as part of the treatment for SAH to prevent vasospasm and delayed ischaemic injury.
    ° However, data from the international, multicentre, randomised controlled STASH trial (Simvastatin in Aneurysmal Subarachnoid Haemorrhage) published in 2014 suggests that there is no short-term or long-term benefit to using statins in these patients, despite earlier enthusiasm with the idea.
    ○ In 2007, a Cochrane review noted that antiplatelet therapy was associated with a non-significant trend indicating a benefit to outcome in patients at risk of delayed cerebral ischaemia.
    ° Unsurprisingly, this trend was counteracted by a parallel increase
    in haemorrhage.
    ° Therefore antiplatelet agents, in this setting, are restricted to use following endovascular stenting for SAH management.
    ○ The only proven effective treatment in the prevention of delayed cerebral ischaemia is nimodipine.
    ° As a calcium antagonist it is thought to protect against vasospasm
    and there is level 1 evidence that it improves outcome.
    °Every patient with a diagnosis of SAH should be started on nimodipine (60mg every 4 hours) for a course of 21 days.
    ° A side-effect can be systemic hypotension which can be avoided by the
    more frequent administration of half doses. If this does not remedy the situation, the blood pressure should take precedence.
    ○ All of these treatments have been considered in the prevention of delayed cerebral ischaemia. Nimodipine is the only one to have withstood repeated testing with consistent results
How well did you know this?
1
Not at all
2
3
4
5
Perfectly
2
Q
  1. A 29-year-old woman who suffered a blow to the left side of her skull vault with a resulting depressed fracture is awaiting transfer to a tertiary centre. She lost consciousness for approximately 1 minute after the incident. Her GCS is currently 14/15 (E4 V4 M6).
    Which of the following, in isolation, indicates that intubation is essential before
    transfer?
    A Pao2 of 13 kPa an Fio2 of 0.6
    B A discrete and short-lived seizure en route to your hospital
    C Drop in GCS from E4 V4 M6 to E3 V4 M5 in the emergency department
    D An increase in respiratory rate leading to a Paco2 of 4.0kPa
    E Blood in the oropharynx
A
  1. B A discrete and short-lived seizure en-route to your
    hospital
    ○ This patient has suffered a head injury by a mechanism significant enough to cause a depressed skull fracture. This will most probably lead to an evolving brain injury secondary to underlying contusions.
    ○ It is important that she is managed in an appropriate environment, to expedite swift treatment of any complications, and is likely to involve further transfer to a tertiary hospital with on-site neurosurgical care.
    ○ Prior to transfer it is imperative to assess her ability to maintain her physiology such that secondary brain injury is avoided as much as possible. °This includes adequate ventilation via a patent airway, preservation of an appropriate blood pressure [cerebral perfusion pressure (CPP) = mean arterial pressure (MAP) - intracranal pressure (ICP)] and optimisation of cerebral metabolism. The aim is to minimise further rises in ICP and secure brain tissue perfusion following the suspected injury.
    ○ The following are suggested targets during transfer:
    • PaO2 >13 kPa
    • PaCO2 4.5–5.0 kPat MAP >80mmHg
    • Adequate analgesia
    • Sufficient sedation (and therefore intubation) if agitated
    • Treatment of any seizures
    • Normothermia
    • Blood glucose 6–10mmol
    • Optimal cerebral venous drainage – head-up, avoidance of neck ties
    ○ In the scenario given you are asked to choose an instance that would obligate you intubate the patient in order to maintain each target en route. The AAGBI has published guidelines for the safe transfer of head injured patients they include indications that should initiate intubation and ventilation before any journey:
    • Glasgow coma score <8/15
    • Glasgow coma score drop of 2 points in the motor score
    • PaO2 <13kPa with oxygen administration
    • PaCO2 <4.0 or >6.0 kPa
    • Concern regarding laryngeal reflexes
    • Seizure(s) since the injury
    • Bilaterally fractured mandible
    • Significant bleeding threatening the airway
    ○ A PaO2 of 13kPa whilst receiving an FIO2 of 0.6 implies a significant alveolar to arterial gradient. The value for PaO2, however, is acceptable and there is scope to improve it with optimal positioning and increased oxygen administration.
    A drop in GCS from E4 V4 M6 to E3 V4 M5 is a drop of 2 points and significant
    enough to warrant consideration of intubation prior to transfer but guidelines allow
    for individual clinical decision making. Intubation is regarded as essential if 2 points
    are dropped within the motor score.
    An increased respiratory rate leading to hypocapnia in this patient could be as a
    result of pain. If, despite treatment, this continues and reduces further to jeopardise
    cerebral circulation then control of ventilation may be warranted.
    Blood in the oropharynx may be small and resolved or ongoing, potentially
    interfering with ventilation. Clinical examination and judgement are required to
    assess whether this, in isolation, would necessitate intubation.
    Seizures in the period following head trauma imply increased severity of the injury
    and may recur to further increase intracranial pressure and cerebral metabolic
    requirements. All of the options could trigger a decision to secure the airway before
    transfer, but seizure activity makes it essential.
How well did you know this?
1
Not at all
2
3
4
5
Perfectly
3
Q
  1. A 65-year-old man presents to the emergency department with acute central
    chest pain radiating to the back. He has a history of hypertension and smoking.
    The ECG shows evidence of left ventricular hypertrophy and his blood pressure is
    190/100mmHg, heart rate 105 beats per minute. There is a collapsing pulse and an
    early diastolic murmur.
    What is the next most appropriate management step?
    A Commencement of sodium nitroprusside infusion
    B Site an arterial line
    C Arrange urgent aortography
    D Titrate intravenous morphine
    E Arrange transfer to a cardiothoracic centre
A
  1. D Titrate intravenous morphine
    The history and clinical signs are suggestive of aortic dissection with aortic
    regurgitation. Other clinical signs relate to the area of the aorta involved and are summarised in Table 6.3.
    ○ There are a number of risk factors for aortic dissection, including:
    • Hypertension (72% of patients)
    • Smoking
    • Trauma – deceleration and falls from height
    • Aortic surgery/cannulation
    • Vasculitis : Syphilis, Takayama arteritis
    • Collagen disorders : Marfans, Ellerslie Danlos, Turner’s
    ○ There are two different classification systems of which the Stanford system is most widely used. It denotes that dissections involving the ascending aorta are Type A with all others as Type B.
    ○ The priorities are to make an accurate diagnosis, limit the stress on the aortic lumen (by lowering systolic blood pressure and left ventricular contractility) and forming a definitive treatment plan, which may include urgent transfer to a cardiothoracic centre.
    ○ It is particularly important to diagnose Type A dissections (i.e. those involving the ascending aorta) as these are considered surgical emergencies. ○ Non-invasive diagnostic methods have superseded traditional aortography (option C) with CT, transthoracic and transoesophageal echo being the most common modalities employed.
    ○ Transthoracic echocardiography can be performed at the bedside but is
    not able visualise the distal ascending and descending aorta reliably.
    ○ The management steps outlined by the European Society of Cardiology guidance
    • Detailed medical history and physical examination
    • Intravenous line, blood samples, cardiac enzymes
    • ECG, heart rate and blood pressure monitoring (both sides)
    • Pain relief
    • Reduction of systolic blood pressure using beta-blockers/calcium channel
    blockers + additional vasodilators if needed
    • Diagnostic imaging
    • Intensive care level monitoring – right radial arterial line as standard
    ○ Although pharmacological control of systolic hypertension may be required, a large proportion of patients will have pain which may of course exacerbate hypertension.
    ○ Titrated morphine is therefore the most appropriate first step in this scenario. If further blood pressure control is required, beta-blockers are recommended before pure vasodilators such as sodium nitroprusside.
    ○ Attainment of clinical stability and institution of invasive blood pressure monitoring would usually be obtained before transfer to a surgical centre, however planning for this possible eventuality early will ensure timely subsequent management.
    ○ Survival after surgical repair of Type A dissection is 96% and 91% at 1 and 3 years respectively. Complicated Type B aortic dissections may be amenable to treatment with endovascular stents, although some centres are also treating Type A dissections in this manner as well.
    ○ Poor prognostic factors at presentation include:
    • Age >70 years
    • Hypotension, shock or tamponade at presentation
    • Preoperative renal failure
    • Preoperative bleeding/massive transfusion
    • Prior myocardial infarction
    • Abnormal ECG are shown below:
How well did you know this?
1
Not at all
2
3
4
5
Perfectly
4
Q
  1. A 65-year-old woman is recovering from an uneventful total thyroidectomy
    as treatment for a large substernal goitre. On the third postoperative day, she
    becomes progressively more stridulous and wheezy. She is tachypnoeic, confused
    and complaining of circumoral paraesthesia. There is no obvious neck swelling or
    pain.
    What is the most likely cause of her symptoms?
    A Bilateral vocal cord paralysis
    B Tracheomalacia
    C Haematoma
    D Tracheal necrosis
    E Hypocalcaemia
A
  1. E Hypocalcaemia
    ○ It is important to remain vigilant for any signs of respiratory distress after head and neck surgery since progression can be rapid with catastrophic consequences.
    ○ After thyroid surgery, there are a number of complications which can cause respiratory difficulties and an appreciation of the associated signs can help identify them.
    ○ Iatrogenic injury to the recurrent laryngeal nerve resulting in vocal cord damage is a recognised complication following thyroid surgery.
    ° Post-operative symptoms depend on whether both the left and right recurrent laryngeal nerves are involved.
    ° Unilateral injury manifests as a hoarse voice, difficulties phonating and aspiration on swallowing whereas bilateral injuries present acutely following extubation with stridor necessitating reintubation and tracheostomy formation.
    ° Bilateral vocal cord paralysis is not the most likely cause in the above scenario, as the stridor only
    presents after four days. Furthermore, bilateral vocal cord paralysis does not directly cause circumoral paraesthesia or confusion.
    ○ Tracheomalacia is believed to occur as a result of longstanding extrinsic tracheal compression causing a loss of tracheal cartilage rigidity.
    ° Removal of this compressive source (thyroidectomy) may then precipitate life threatening dynamic airway collapse.
    ° It is a very rare complication and does not explain the confusion and paraesthesia in the above scenario.
    ○ Post-operative haemorrhage is a well recognised complication following thyroid surgery and can result in a rapidly expanding haematoma compromising airway patency.
    ° The haematoma usually presents as a large, tense and immobile swelling
    under the wound, which will have to be re-opened at the bedside if there is
    impending airway obstruction.
    ° The majority of bleeds occur within 24 hours and presenting symptoms can include stridor, dyspnoea, neck pain, dysphagia and confusion.
    ° Although an important differential to consider, it is not the most likely
    diagnosis in the above case due to the normal neck examination and lack of pain.
    ° Symptom occurrence on day four postoperatively and the presence of circumoral paraesthesia is also not typical.
    ° The blood supply to the upper trachea is primarily from small branches of the inferior thyroid artery and life threatening tracheal necrosis due to excessive cautery near the trachea has been described.
    ○ Tracheal disruption is unlikely to be causing
    the symptoms in the case described since there is no subcutaneous emphysema (formed from the tracheal air leak).
    ° Furthermore, stridor, confusion and paraesthesia are not usually associated with this very rare complication.
    ○ Hypocalcaemia is the most common complication following thyroidectomy and the most likely cause of the clinical picture described.
    ° Since the parathyroid glands are located on the posterior surface of the thyroid gland, these can be damaged or devascularised following surgery to this area.
    ° The fall in calcium levels generally occurs within 24–48 hours post-operatively and can be sufficient to produce symptoms.
    ° Hypocalcaemia directly increases neuromuscular excitability and
    many of the clinical manifestations stem from this underlying problem.
    ° The stridor described in the above case is due to laryngospasm which is an exaggeration of the normal glottic closure reflex.
    ° Circumoral paraesthesia and bronchospasm also arise as a consequence of neuromuscular irritability
How well did you know this?
1
Not at all
2
3
4
5
Perfectly
5
Q
  1. A 36-year-old woman with an impacted food bolus needs to go to theatre
    imminently. She has been unable to swallow her saliva for 24 hours. On inquiring
    about her anaesthetic history she reports collapsing due to a severe allergic
    reaction under anaesthesia, but she is unsure which agent was responsible. There
    are no notes available, nor relatives to elaborate on the history.
    Which of the following should you avoid as the most likely causative agent?
    A Rocuronium
    B Latex
    C Morphine
    D Chlorhexidine
    E Gelofusine
A
  1. A Rocuronium
    ○ This patient’s limited anaesthetic history raises the suspicion of a previous episode of anaphylaxis.
    ○ Without prior records available it is prudent to avoid agents most likely
    to cause such a reaction.
    ○ Anaphylaxis is an immune reaction that is triggered by hypersensitivity to an antigen, e.g. the β lactam ring found in some antibiotics. It results in IgE antibody production and a subsequent IgE-antigen mediated cascade of events.
    ° This leads to the widespread release of inflammatory mediators such as histamine, leukotrienes and prostaglandins.
    ° The reaction results in an increase in vascular permeability, bronchial hyper-reactivity and subsequent circulatory compromise that can be fatal (10% of those reported to the UK Medicines Control Agency).
    ○ Similar, and often indistinguishable, reactions may occur that do not involve IgE
    release in response to an antigen. They manifest secondary to direct histamine release or activation of the complement pathway by other means. ° They are known as anaphylactoid reactions. An example of which could be initiated by morphine which acts directly on mast cells to cause histamine release.
    ○ The culture of reporting anaphylactic reactions is variable between countries and thus the frequency of its occurrence (based on information from Australia and France) ranges from 1 in 10 000 to 1 in 20 000. The 6th National Audit Project (Perioperative Anaphylaxis) may help determine the incidence of anaphylaxis in the UK, which is currently unknown.
    ○ The following table (Table 6.4) lists the most commonly known triggers for
    anaphylaxis and their proposed incidence when associated with anaesthesia.
    ° As muscle relaxants are reported to be the agents with the highest risk of triggering anaphylaxis, rocuronium should be avoided in this scenario if at all possible.
    ° If the use of a muscle relaxant is necessary, using a benzyl-isoquinolinium instead of an aminosteroid may reduce the risk as they are less associated with such a reaction.
    ° To further avoid histamine release, and therefore the possibility of an anaphylactoid reaction, cisatracurium may be the best option.
    ○ The remaining agents can also be associated with anaphylaxis. Further modifications to the anaesthetic, such as fentanyl instead of histamine-producing morphine or iodine in place of chlorhexidine and avoidance of all colloids, can be simple enough to make.
    The majority of theatres are now run as ‘latex-free’ or can easily be made so these days.
How well did you know this?
1
Not at all
2
3
4
5
Perfectly
6
Q
  1. A 34-year-old man presents for laparoscopic excision of his left adrenal gland for phaeochromocytoma. During your preoperative assessment, he tells you that he has been taking medication for blood pressure for about a month. Which of the following is most likely to indicate that he is prepared for surgery?
    A Good exercise tolerance, but a history of dizziness on standing
    B Lack of a history of palpitations, and a normal ECG
    C A normal echocardiogram, and chest X-ray
    D History of dizziness on standing, a 5-minute ECG with no premature
    ventricular complexes, and nasal congestion
    E Several blood pressure recordings of <160/90mmHg
A
  1. D History of dizziness on standing, a 5-minute ECG with no premature ventricular complexes (PVCs), and nasal congestion
    ○ Phaechromocytomas, although rare in clinical practice are more common in exams. This secreting tumour is named a chromaffinoma, because of its derivation from chromaffin cells which evolve from the neural crest to make up the normal sympathetic system.
    ○ The classical clinical syndrome of severe hypertensive crises
    accompanied by headache, sweating, palpitations and anxiety, with resolution afterwards, is variable and depends mainly on the secretory properties of the tumour.
    ○ Most secrete noradrenaline; with some producing both noradrenaline and adrenaline and a few may also secrete active peptides such as adrenocorticotrophic hormone (ACTH), calcitonin, vasoactive intestinal peptide (VIP) and somatostatin also.
    ○ Tumours are 90% adrenal and 10% extra-adrenal, known as paragangliomas.
    ○ The full range of imaging techniques is used for their identification, with functional PET scanning in some centres. M-iodobenzylguanidine (MIBG) isotope uptake scans are useful to identify tumour foci and locate extra-adrenal or secondary deposits.
    ○ Preoperative assessment and preparation is of paramount importance, and focuses on assessment for pathology associated with the tumour, namely end-organ damage caused by hypertension, and pharmacological suppression.
    °With pharmacological suppression the classic target criteria are:
    • Blood pressure <160/90mmHg
    • Postural hypotension, but not severe (<80/45mmHg)
    • ECG free from ST/T wave changes for 7 days
    • No greater than one premature ventricular contraction on ECG every 5 minutes
    • Nasal congestion
    ○ Agents used include the non-specific α-blocker phenoxybenzamine (which due to α2 blockade also causes tachycardia, and therefore must be given with a β-blocker).
    ○ Selective α1 blockers, such as doxazosin can now be used alone.
    ○ If β-blockade is required, a stable a block has to be established first to prevent the loss of β2 vasodilatation, and therefore increased hypertension.
    ○ Some of the stems in this question look for signs of α blockade.
    ° These may include postural hypotension, and nasal congestion.
    ° Lack of cardiac irritability feature in B and D, and are also reassuring, but the normal ECG reading in B cannot exclude ectopic beats.
    ° The normal chest X-ray and echocardiogram in C are reassuring, but cannot exclude acute physiological changes seen with this condition.
    ° Repeated blood pressure readings <160/90mmHg (E) are also reassuring about good blood pressure control, but the stem with both reassuring symptoms and physiological investigations is option D.
How well did you know this?
1
Not at all
2
3
4
5
Perfectly
7
Q
  1. A 45-year-old man is admitted to the surgical ward with a fever, toothache and neck discomfort. Whilst waiting for surgery you are called to his bedside as he is more breathless and complaining of substernal pain. On examination he is hypotensive and there is tender, ’woody‘ induration of his neck. On auscultation you hear a pericardial rub.
    Which investigation is most appropriate to guide management in this scenario?
    A Cervical and chest ultrasound
    B Cervical and chest computed tomography
    C Cervical and chest magnetic resonance imaging
    D Cervical and chest radiograph
    E Echocardiogram
A
  1. B Cervical and chest computed tomography
    ○ The above case describes Ludwig’s angina which is an aggressive, rapidly spreading “woody” cellulitis of the submandibular space, commonly arising from an infected molar tooth.
    ○ There is a lack of lymphadenopathy since the typically polymicrobial
    infection spreads along fascial planes as opposed to the lymphatic system.
    ○ Two life-threatening complications of Ludwig’s angina are upper airway obstruction and descending necrotising mediastinitis.
    ○ Sufferers are at risk of airway obstruction due to posterior infective extension and tongue distension with posterior displacement.
    ○ Descending necrotising mediastinitis describes the spread of infection from neck to mediastinum via contiguous fascial planes which is promoted by gravity and the negative intrathoracic pressure.
    ○ Since the disease is rare and early symptoms often nebulous, diagnosis and treatment can be delayed with fatal consequences.
    ○ Computed tomography is the imaging modality of choice for acute deep-seated neck infections and the correct answer to the above scenario. Imaging the neck allows a rapid assessment of the depth of involvement as well as the presence of abscesses which may be amenable to surgical drainage.
    ○ Chest imaging provides confirmation and allows an assessment of the extent of mediastinal involvement which is important for surgical planning.
    ○ Since a pericardial rub was heard in the above scenario, computed tomography will also be useful in assessing for secondary pericardial involvement and the presence of an effusion.
    ○ Magnetic resonance imaging does provide excellent soft tissue resolution and diffusion weighted imaging can help delineate complex fluid collections. ° This imaging modality is particularly useful for infections involving the retropharyngeal space where extension into the spinal column is suspected. However, it is more time consuming than computed tomography and patients may feel claustrophobic during scanning. Patient compatibility also needs to be assessed. In the above scenario where the airway patency can deteriorate rapidly and an early diagnosis and treatment plan is needed, computed tomography is more appropriate.
    Cervical ultrasound can be useful in characterising soft tissue masses and collections
    but is unable to penetrate bone or air filled structures. It is also operator dependent
    and not as accurate as computed tomography in assessing the extent of mediastinal
    involvement.
    Plain radiography is easily accessible but is of little value in planning the
    management of descending necrotising fasciitis complicating Ludwig’s angina.
    ○ A lateral cervical radiograph can highlight pretracheal gas bubbles and a loss of the normal lordosis, whereas a chest radiograph may show a widened mediastinum and an enlarged cardiac silhouette if there is mediastinitis or a pericardial effusion respectively. Computed tomography however provides a much more accurate picture of the severity of the infection.
    ○An echocardiogram can provide information regarding the extent of the pericardial effusion and whether it is affecting cardiac function. Echocardiography is not the most appropriate investigation to plan management since it provides no information on the degree of cervical involvement or whether there are any collections amenable to drainage
How well did you know this?
1
Not at all
2
3
4
5
Perfectly
8
Q
  1. A 35-year-old cyclist suffered a severe traumatic brain injury with a large subdural haematoma and an associated C2–C3 cervical spine fracture. He is comatose and apnoeic, with neurosurgeons confirming that he is not a candidate for surgery due to poor prognosis. Confirmation of brainstem death is underway, with examination of cranial nerves just being completed.
    What is the next most appropriate test that will support the neurological diagnosis of death?
    A Apnoea testing
    B Somatosensory evoked potentials
    C No further tests necessary
    D A second neurological examination of the cranial nerves
    E Electroencephalogram
A
  1. E Electroencephalogram
    ○ The patient fulfils the prerequisites for brainstem testing because he has suffered irreversible brain injury and he is in an apnoeic coma.
    ○ The neurological confirmation of death consists of cranial nerve II – XI examination and apnoea testing performed by two doctors at two different times.
    • At the end of each set of cranial nerves examinations an apnoea test occurs.
    • In a patient with a high cervical spine injury, apnoea might not be due to a central cause but due to spinal cord injury, therefore ancillary tests are employed to confirm de the diagnosis.
    • Electroencephalogram (EEG) is the most widely used and validated assessment in this circumstance.
    • The second battery of brainstem tests cannot be performed in isolation without the apnoea testing; therefore an EEG is the next most appropriate step to support the diagnosis of death by neurological criteria. • Somatosensory evoked potentials are used for monitoring of depth of anaesthesia and play no part in the diagnosis of death.
How well did you know this?
1
Not at all
2
3
4
5
Perfectly
9
Q
  1. A 35-year old man for elective ankle surgery is to have an ultrasound guided popliteal nerve block.
    What is the most frequently used combination of ultrasound view and needle visualisation for this nerve block?
    A Short-axis view with in-plane needle approach
    B Long-axis view with out-of-plane needle approach
    C Short-axis view with out-of-plane needle approach
    D Long-axis view with in-plane needle approach
    E Any of the above combinations
A
  1. A Short-axis view with in-plane needle approach
    ○ The use of ultrasound (US) in regional anaesthesia has significantly increased in the recent years.
    ○ Choosing the correct US view and needle orientation is essential for successful and safe nerve block.
    ○ When scanning nerves the structures viewed by US beam will either be in a short-axis view or long-axis view.
    ○ In the short-axis view, the nerves and the blood vessels are visualised in section (sliced across their diameter), nerves are easier to find, and the US image is relatively stable making this view ideal for introducing a needle.
    ○ In the long-axis view, however, the nerves and blood vessels are visualised longitudinally along their length (demonstrating a tube like structure) making the US image produced unstable and not ideal for needle insertion.
    ○ When introducing the needle, it can be passed either along the long-axis of the US beam (in-plane) or across the short-axis of the beam (out-of-plane). ○ With an in-plane approach, the needle is visualised entirely throughout the block and produces good views of needle-nerve proximity. Therefore this is the safest approach.
    ○ With an out-of-plane technique, the needle crosses the US beam as a bright dot and the accurate location of the needle tip is uncertain and it could be advanced in unwanted tissue, making this approach less safe for needle insertion. However, anaesthetists the out-of-plane approach is ideal when inserting catheters as it allows parallel advancement of the catheter along the long-axis of the nerve as it exits the tip of the needle (Figure 6.1).
    ○ In this example, the combination of short-axis view and in-plane needle visualisation is the safest approach for the above reasons.
How well did you know this?
1
Not at all
2
3
4
5
Perfectly
10
Q
  1. You are called to the emergency department to assess a young woman that was rescued from a house fire following a gas leak after being trapped confined in a room. She is awake, with normal observations but suffered 10% body surface area (BS) partial thickness burns over her arms and face. You are asked to transfer her to the nearest burns unit that is 2 hours away. On examination she has singed nasal hair, a normal airway and no change in voice. Burns resuscitation is underway with intravenous fluids and analgesia.
    What is the next step in ensuring her safe transfer?
    A Add the operating department practitioner to your transfer team
    B Full monitoring including invasive blood pressure measurement
    C Prepare difficult airway equipment for the transfer
    D Prepare Intubating equipment and drugs
    E Elective intubation of the patient
A
  1. E Elective intubation of the patient
    ○ Inhalational injury is the aspiration of heated gases, hot liquids, steam, or noxious substances of incomplete combustion.
    ○ It can be categorised as:
    • Upper airway thermal injury – supraglottic burns causing stridor, a change in voice quality or uvular oedema
    • Lower airway thermal injury – infraglottic burns most commonly by noxious by-products of incomplete combustion leading to dyspnoea, wheeze and secretions
    • Noxious gases injury – including inhalation of carbon monoxide and cyanide
    ○ This patient has a high risk of inhalation injury due to an enclosed space fire with significant burns to the face. The onset of airway oedema is often unpredictable, but fluid resuscitation is likely to worsen any impending oedema, while the relatively long duration of transfer indicates the need to have a secure airway during transfer.
    ○ Therefore it is appropriate to plan elective intubation of the patient in controlled circumstances with senior support, a difficult airway trolley and skilled assistance.
    ○ Adding a competent team member to the transfer is reassuring and can help should complications arise during transfer, but it is often impractical. All transfers should have full monitoring, including ECG, pulse oximetry and non-invasive blood pressures, but invasive blood pressure monitoring is only indicated if you anticipate cardiovascular instability or it is required to guide ongoing therapy.
    ○ Availability of difficult airway equipment is necessary once elective intubation has been decided, and devices such as video laryngoscopes are useful to have when a patient is being transferred. However, the most appropriate approach would be to ensure a secure airway prior to transfer.
How well did you know this?
1
Not at all
2
3
4
5
Perfectly
11
Q
  1. You are called to the emergency department to assess a young woman that was
    rescued from a house fire following a gas leak after being trapped confined in a
    room. She is awake, with normal observations but suffered 10% body surface area
    (BS) partial thickness burns over her arms and face. You are asked to transfer her
    to the nearest burns unit that is 2 hours away. On examination she has singed nasal
    hair, a normal airway and no change in voice. Burns resuscitation is underway with
    intravenous fluids and analgesia.
    What is the next step in ensuring her safe transfer?
    A Add the operating department practitioner to your transfer team
    B Full monitoring including invasive blood pressure measurement
    C Prepare difficult airway equipment for the transfer
    D Prepare Intubating equipment and drugs
    E Elective intubation of the patient
A
  1. E Elective intubation of the patient
    Inhalational injury is the aspiration of heated gases, hot liquids, steam, or noxious
    substances of incomplete combustion. It can be categorised as:
    t Upper airway thermal injury – supraglottic burns causing stridor, a change in
    voice quality or uvular oedema
    t Lower airway thermal injury – infraglottic burns most commonly by noxious by-
    products of incomplete combustion leading to dyspnoea, wheeze and secretions
    t Noxious gases injury – including inhalation of carbon monoxide and cyanideThis patient has a high risk of inhalation injury due to an enclosed space fire with
    significant burns to the face. The onset of airway oedema is often unpredictable, but
    fluid resuscitation is likely to worsen any impending oedema, while the relatively
    long duration of transfer indicates the need to have a secure airway during transfer.
    Therefore it is appropriate to plan elective intubation of the patient in controlled
    circumstances with senior support, a difficult airway trolley and skilled assistance.
    Adding a competent team member to the transfer is reassuring and can help
    should complications arise during transfer, but it is often impractical. All transfers
    should have full monitoring, including ECG, pulse oximetry and non-invasive blood
    pressures, but invasive blood pressure monitoring is only indicated if you anticipate
    cardiovascular instability or it is required to guide ongoing therapy. Availability of
    difficult airway equipment is necessary once elective intubation has been decided,
    and devices such as video laryngoscopes are useful to have when a patient is being
    transferred. However, the most appropriate approach would be to ensure a secure
    airway prior to transfer.
How well did you know this?
1
Not at all
2
3
4
5
Perfectly
12
Q
  1. A 72-year-old man has been on the intensive care unit after being treated for an
    infective exacerbation of his chronic obstructive pulmonary disease. He has been
    mechanically ventilated for 5 days and has acceptable gas exchange. He has been
    weaned to pressure support ventilation requiring 12cmH2O inspiratory support
    and 5 cmH2O of positive end-expiratory pressure (PEEP) with an inspired oxygen
    concentration of 0.35. He is currently obeying commends.
    How would you best assess his suitability for extubation?
    A Change the patient to continuous positive airway pressure (CPAP) and assess
    ventilation and cardiovascular parameters for 30 minutes
    B Reduce the pressure support gradually over the next 48 hours by 2 cmH2O per
    12 hours and assess ventilation and cardiovascular parameters
    C Reduce the inspired oxygen fraction to 0.25 and repeat an arterial blood gas 30
    minutes later
    D Repeat a chest radiograph to ensure resolution of his consolidative process
    E Assess the patient’s sputum production and send a repeat sample for
    microscopy to ensure clearance of the infective process
A
  1. A Change the patient to continuous positive airway
    pressure (CPAP) and assess ventilation and cardiovascular
    parameters for 30 minutes
    The majority of patients who receive mechanical ventilation have acute respiratory
    failure in the postoperative period, pneumonia, congestive heart failure, sepsis,
    trauma or acute respiratory distress syndrome (ARDS). Respiratory muscle
    weakness may not be a contributing factor to their respiratory failure and once
    the acute pathophysiological problem is resolved, invasive ventilation may be
    downgraded and patients extubated. The duration of mechanical ventilation is
    often unnecessarily prolonged in the setting of a short period of ventilator support
    (less than 7 days) with the weaning process accounting for up to 50% of the total
    ventilation time. A delay of 48 hours in extubation results in an increased risk of
    extubation failure, ventilator acquired pneumonia, thromboembolic disease, longer
    intensive care and hospital stay and increased mortality.
    Weaning involves progression from a controlled mode of ventilation to a support
    mode and then reduction of support delivered until a trial of readiness for
    extubation. This trial is termed a spontaneous breathing trial (SBT).
    Typical readiness criteria for attempted weaning include:
    t Improvement in the underlying condition that caused the respiratory failure
    t Pulmonary: fractional inspired oxygen Ratio (PFR) of more than 200 with a
    positive end-expiratory pressure (PEEP) of 5cmH2O
    t Haemodynamic stability
    t No electrolyte, metabolic, haematological or nutritional deficits
    t Neurologically appropriate with cough and gag reflexes present
    Once deemed suitable, a SBT may be initiated with minimal pressure support such
    as 5cmH2O, CPAP or a T-piece or tracheostomy mask (no PEEP). A SBT should be attempted for a minimum of 30 minutes but should be terminated and deemed
    unsuccessful if:
    t The respiratory rate remains above 35 breathes per minute for 5 minutes
    t Oxygen saturations of less than 90%
    t Heart rate increases to over 140 beats per minute
    t Systolic blood pressure >180mmHg or <90mmHg
    t Panic or diaphoresis
    The following classification of the results of the spontaneous breathing trial may be
    applied:
    t Simple: successful first trial followed by extubation
    t Difficult: up to three spontaneous trials but discontinuation of ventilation within
    7 days
    t Prolonged: more than three unsuccessful trials or more than 7 days of mechanical
    ventilation
    10–20% of ventilated patients may have prolonged weaning and in-hospital
    mortality is increased in this group. In patients who fail a SBT, the strategy is to
    reduce the support the patient is receiving and try again. A period of rest between
    SBTs is advocated of 24–48 hours. Gradual reductions in the pressure support
    by 2–4cmH2O per 24 hours or a short SBT period every hour with increasing the
    duration are both advocated.
    Even assessing in a careful manner such as described above, 10–15% of extubations
    fail, necessitating re-intubation. If this occurs the mortality rate in this group is
    increased.
    In the patient above, the criteria for initiating a SBT is met and if successful, a trial of
    extubation is warranted. In this patient with COPD, a greater-than-average sputum
    production and poorer gas-exchange may have been present prior to the acute
    infection and must be accepted in order to avoid the complications of on-going
    mechanical ventilation.
How well did you know this?
1
Not at all
2
3
4
5
Perfectly
13
Q
  1. You are asked to review a 72-year-old man who was admitted to your intensive care unit 6 hours ago following elective coronary artery bypass grafts. He is haemodynamically stable with no evidence of end organ hypoperfusion. The concern is that he has been slowly bleeding into his drains (total 570mL since theatre) and has slow oozing through his sternotomy wound and around his lines and drains. His core temperature is 36.2°C and pH 7.32. An urgent full blood count and clotting tests were sent 30 minutes ago and the results have just come back and show: Haemoglobin concentration 78g/L; platelet count 102 ×109/L; INR 1.4; aPTTr 1.6; fibrinogen 1.8g/L; and ionised calcium 0.9mmol/L. The patient is on long-term aspirin 75mg daily (not stopped for surgery). He received heparin in theatre that was reversed with protamine. He also received a single dose of 1g tranexamic acid.
    Given this information the most appropriate treatment strategy is:
    A 1 unit packed red blood cells (pRBC) + 1 pool of platelets + 15mL/kg fresh
    frozen plasma (FFP) + 1 dose of cryoprecipitate
    B 20mmol of calcium chloride + protamine + 1 pool of platelets + tranexamic
    acid
    C 2 units packed red blood cells (pRBC)
    D 20mmol of calcium chloride + 1 pool of platelets + 15mL/kg fresh frozen
    plasma (FFP)
    E Perform a thromboelastogram
A
  1. D 20mmol of calcium chloride + 1 pool of platelets +
    15mL/kg fresh frozen plasma (FFP)
    ○ In order to form effective blood clots a patient needs an adequate number of functioning platelets, adequate levels of all the clotting factors, an adequate haematocrit, an adequate level of ionised calcium, a relatively normal pH and an absence of significant hypothermia.
    ○ The critical levels of these variables cannot be defined and are mutually dependent.
    ○ The clinical scenario described suggests that there is ongoing bleeding due to a coagulopathy rather than a failure of surgical haemostasis.
    ○ The temperature, pH, haematocrit and platelet count are acceptable.
    °However, the patient has been receiving long term antiplatelet therapy and has been on cardiopulmonary bypass, thus, in the absence of a platelet function test it is reasonable to deduce that platelet transfusion is warranted to correct the coagulopathy.
    °The clotting tests suggest there is a consumptive and /or dilutional component to this coagulopathy.
    ○ Given the degree of abnormality, a dose of FFP should elevate the levels of all factors, including fibrinogen, without the need to give additional cryoprecipitate.
    ○ Administration of FFP and platelets is likely to result in a further drop in ionised calcium, it would be prudent to administer a replacement dose. Given the scenario and timings, a further dose of protamine is likely to result in an anti-coagulant effect. In the absence of evidence for hyperfibrinolysis, a second dose of tranexamic acid is not indicated at this stage.
    ○ The threshold for pRBC transfusion in this context is <70g/L.
    ○ A thromboelastogram would refine the diagnosis further and repetition after intervention guide further therapy. This is a common practice in many centres but not universal.
How well did you know this?
1
Not at all
2
3
4
5
Perfectly
14
Q
  1. A 60kg, 55-year-old woman has been admitted to the intensive care unit with severe community acquired pneumonia. Two days later she develops worsening hypoxaemia with new bilateral infiltrates on chest radiography. She is currently ventilated with the following settings:
    • Fio2 1.0
    • Inspiratory pressure (Pinsp) 35 cmH2O
    • Positive end expiratory pressure (PEEP) 12cmH2O
    • Inspiratory:expiratory (I:E) ratio 1:1
    • Tidal volume (Vt) 250mL
    An arterial blood gas reveals results shown in Table 6.1.
    Based on current evidence, which of the following would be an appropriate next
    step to improve her oxygenation and reduce mortality?
    A Extracorporeal membrane oxygenation
    B Prone positioning
    C Inhaled nitric oxide
    D High frequency oscillation ventilation
    E Increase Pinsp
A
  1. B Prone positioning
    ○ The worsening hypoxaemia, new bilateral radiology infiltrates and low PaO2:FIO2 (P:F ratio) within one week of the onset of severe pneumonia suggests acute respiratory distress syndrome (ARDS).
    ○ ARDS is an acute, diffuse inflammatory lung syndrome that results in respiratory failure. The 1994 American-European Consensus Conference definition of ARDS has now been superseded by the 2012 Berlin Definition (Table 6.5).The cause of ARDS in this patient is severe pneumonia, which is a direct (or pulmonary) cause. Other direct causes of ARDS include aspiration, lung contusions and inhalational injury. Indirect (non-pulmonary) causes include sepsis, trauma, pancreatitis and burns.
    ○ The pathophysiology of ARDS is complex and involves the interplay of various body systems. A simplified view of this pathogenesis is presented here but this is an area of ongoing exploration
  2. Exudative phase: Alveolar capillary membrane disruption resulting in leakage of protein rich fluid. Inflammatory cells (e.g. neutrophils) infiltration forming exudate.
  3. Proliferative phase: Proliferation of abnormal type II alveolar cells and inflammatory cells. There is a resultant dysfunction in surfactant with decreased pulmonary compliance.
  4. Fibrotic phase: Infiltration with fibroblasts replacing alveolar cells and ducts resulting in marked reduction in pulmonary compliance.
  5. Restoration phase: Slow and incomplete repair of pulmonary architecture.
    The management of ARDS can be subdivided as below:
    ○ General (‘FLATHUGS’)
    • Feeding – early nutrition
    • Lines – as per catheter-related blood stream infection bundle
    • Analagesia – adequate to maintain patient comfort, avoid under or oversedation
    • Thromboprophylaxis – consider non-pharmacological and pharmacological
    • Hydration – FACCT trial (2006) did not show a difference in fluid therapy guided by pulmonary artery flotation catheter versus central venous catheter
    • Ulcer prophylaxis – according to local protocol and review daily
    • Glycaemic control – no definitive evidence for tight glycemic control, aim for glucose <10mmol/L
    • Sedation/Spontaneous breathing trial – consider daily sedation holds and breathing trials
    ○ Mechanical ventilation (based on ARDSnet mechanical ventilation protocol summary)
    • Tidal volume 6mL/kg : ARMA study (2000) investigated 12mL/kg versus 6mL/kg in acute lung injury, lower tidal volumes resulted in improved outcomes
    • Plateau pressures (Pplateau)<30 cmH20
    • Permissive hypercapnia, aim for pH >7.3
    • PEEP; ALVEOLI trial (2004) demonstrated an absence of data proving superiority of lower or higher PEEP for survival

○ ‘Rescue’ therapies for refractory hypoxaemia
° Prone position
-Prone positioning is based on the theory of recruiting areas of lung that are non-dependent in the supine position, leading to reduced ventilation-perfusion mismatching.
-There are additional benefits of improved secretion clearance and increased homogeneity of ventilation due to decreased lung deformation by mediastinal structures.
-There are potential adverse effects such as line or endotracheal tube displacement, reduced preload and functional restriction in cardiac contraction, pancreatitis, raised intracranial pressure and pressure related nerve damage.
-The process itself needs to be meticulously performed with adequate numbers of staff.
-PROSEVA (2013) was a landmark prospective, multicenter randomised control trial investigating early prone positioning in moderate to severe ARDS. It suggests benefit in terms of oxygenation and mortality. Previous studies appeared to show improved oxygenation, but no clear mortality benefit.
° Inhaled nitric oxide
-Nitric oxide (NO) is known to cause pulmonary vasodilatation and hence improve pulmonary blood flow.
-The inhaled route delivers NO selectively to ventilated lung units and hence improves oxygenation.
-Although inhaled nitric oxide improves oxygenation, there does not appear to be a mortality benefit.

° Extracorporeal membrane oxygenation
ECMO involves insertion of large cannulae into central vessels. It is similar to a simple cardiopulmonary bypass circuit. Blood leaves a central vessel and is pumped around a circuit through a membrane oxygenator to allow gas exchange, then returned to the patient via a central vessel. As oxygenation is predominantly achieved through the extracorporeal circuit, ultra low tidal volumes can be used to ventilate the patient minimising ventilator associated lung injury. ECMO requires systemic anticoagulation, carrying a risk of bleeding.
The CESAR trial (2009) was a multicentre randomised control trial investigating conventional management or referral to consideration for treatment by ECMO in severe potentially reversible respiratory failure. It concluded that referral to a tertiary respiratory centre for consideration of ECMO resulted in improved survival.
It is unclear what proportion of this benefit is attributed to optimum conventional ventilation in a tertiary referral centre.

○ High frequency oscillation ventilation
°HFOV works on the principle of high frequency (120–600 breaths/min) oscillation around a continuous high distending airway pressure. This results in lower tidal volumes, approximating at 3mL/kg. The OSCAR (2013) and OSCILLATE (2013) trials were two multicentre randomised control trials investigating the role of HFOV in ARDS. The failed to show a benefit and possibly showed harm with HFOV compared to conventional ventilation. At present, HFOV cannot be recommended in refractory hypoxemia secondary to ARDS in adults.
Of the options given in this question, based on current evidence the options for refractory hypoxaemia in ARDS appear to be prone positioning and ECMO. Given that ECMO remains controversial and requires a specialist centre, prone positioning
would be the most appropriate option in this patient. If an option was given for ‘referral to a tertiary respiratory centre’, that would also be appropriate.
○ Pharmacological
No proven mortality benefit but many have been trialed including surfactant replacement therapy, glucocorticoids, and β-adrenoceptor agonists

How well did you know this?
1
Not at all
2
3
4
5
Perfectly
15
Q
  1. A 58-year-old man is brought in by ambulance following a house fire in an
    enclosed area. He is confused with a GCS14/15.
    On examination he has singed facial hair with voice changes. He is noted to have
    partial thickness burns to the front of his torso, bilateral palms and palmar aspect
    of upper limbs. His body weight is 70kg.
    According to the Parkland formula his estimated fluid requirement in the first 8
    hours following his burn is:
    A 7560mL
    B 4850mL
    C 4620mL
    D 3910mL
    E. 780mL
A
  1. E 3780mL
    This patient has sustained a significant thermal injury with evidence of inhalational injury.
    ○ Significant burns cause a profound systemic inflammatory response
    syndrome and early aggressive management is paramount. Mortality from major burns is in the order of 10–20% with multiorgan failure and sepsis being leading causes.
    ○ Management should follow ALTS guidelines, especially where the mechanism is unknown. During the primary survey, early intubation is advised where airway compromise or significant inhalational injury is suspected. A rapid sequence induction is advised and intubation performed with an uncut cuffed endotracheal tube; ideally size 8 or larger to aid assessment of the airway via bronchoscopy.
    ○ Suxamethonium is considered safe in the first 24 hours following injury, an exaggerated hyperkalaemic response may occur after this time frame.
    ○ As part of the ‘Breathing’ assessment, carbon monoxide poisoning should be excluded.
    ○ In this case the confusion at presentation may be an early sign and an arterial blood gas should be done urgently.
    ○ Normal carbon monoxide levels can be up to 10% in smokers and a level greater than 20% raises the suspicion of significant inhalation injury and carbon monoxide poisoning. It is important to note that pulse oximetry overestimates SpO2 in the presence of carbon monoxide.
    °Therefore the saturations of 100% in this case should be corroborated with arterial gas analysis via co-oximetry.
    °High-flow oxygen decreases the half-life of carbon monoxide from 4 to 1 hours, and should be administered empirically until carboxyhaemoglobin (HbCO) levels are attained.
    ○ Another point of concern in this patient as part of the ‘Breathing’ assessment is the anterior torso burn.
    °The chest wall should be examined for evidence of circumferential burn which may require early escharotomies.
    °There is evidence to support that, where possible, these should be done in specialist burns centres.
    °The focus of this question is on the assessment of circulation.
    °As the burns surface area affects the management of fluid resuscitation, this must be calculated at this stage.
    ○ The body surface area (BSA) takes into account partial and full thickness burns and can be calculated using the ‘rule of 9s’. In this patient the burn to the anterior torso represents 18% BSA and bilateral palmar surfaces of upper limbs represent a further 9% (i.e. 2 x 4.5%); the total BSA is 27% (Figure 6.2).
    ○ The Parkland formula is widely used in the UK for calculation of fluid resuscitation with warmed crystalloid. It calculates the fluid requirement for the first 24 hours, from the time of injury, not the time of first presentation.
    ° Parkland formula for fluid requirement = 4mL/kg/% BSA
    Fluid requirement in this patient = 4mL x 70kg x 27% = 7560mL
    ° According to the Parkland formula, half of this volume should be given in the first 8 hours making 3780mL correct.
    °The Parkland formula is an estimation and fluid therapy should be guided by clinical and physiological parameters; there are detrimental consequences of both under and over resuscitation with fluids.
    ○ Management of the burn itself with early decontamination and ensuring
    normothermia are important early considerations.
    ○ Antibiotic use should be reserved to where there is a strong clinical suspicion of active infection, there appears to be little evidence for prophylactic antibiotics.
    ° There are burns specific criteria for diagnosing sepsis which can guide clinical decision making.
    ° This patient represents a BSA greater than 10%, with burns to hands and a possible inhalational injury mandating discussion and transfer to a regional burns centre.
    ○ The British Burns Association criteria for referral to a burns centre are shown in Table 6.6 below.
How well did you know this?
1
Not at all
2
3
4
5
Perfectly
16
Q
  1. A male motorcyclist of unknown age has been transferred to hospital after having a high-speed accident. The paramedics report states that the patient is unresponsive,has chest, abdominal, pelvic injuries and a traumatic right leg amputation currently secured with a tourniquet. The respiratory rate is 10 breaths per minute, there is a weak carotid pulse and the Glasgow coma score is 3. The trauma team members are present and you decide to prepare to intubate the patient.
    What technique is most appropriate?
    A Modified rapid sequence with rocuronium 1mg/kg and midazolam 0.05mg/kg
    B Rapid sequence induction with thiopentone 3mg/kg and suxamethonium
    1mg/kg
    C Modified rapid sequence induction with midazolam 0.05mg/kg, fentanyl
    1–3µg/kg and rocuronium 1mg/kg
    D Modified rapid sequence induction with propofol 1–2mg/kg, fentanyl 1–3µg/kg and rocuronium 1mg/kg
    E Modified rapid sequence with ketamine 2mg/kg, fentanyl 1–3µg/kg and
    rocuronium 1mg/kg
A
  1. E Modified rapid sequence with ketamine 2mg/kg,
    fentanyl 1–3µg/kg and rocuronum 1mg/kg
    ○ Anaesthesia service representation in the trauma team may come from the anaesthetic department or intensive care unit (ICU). The answer to this question is often the technique in which the clinician has the most experience with because a stressful situation is not the ideal time to be trying out novel methods; however, consideration of the different options is still important.
    Your responsibility is to secure the airway while your colleagues simultaneously manage some of the other issues. Your choice of induction technique is vital with some of the popular options listed above. The considerations include:
    t Speed of induction:
    – The patient has been obtunded for some time and therefore performing
    an induction designed to progress from unconsciousness to endotracheal
    intubation in as rapid a time as possible is less vital. The classic rapid sequence induction of thiopentone and suxamethonium causes vasodilation, reflex tachycardia, decreased myocardial contractility and suxamethonium causes a transient rise in intracranial pressure.
    • Cardiovascular stability is the most important aspect of this situation. Therefore the use of anaesthetic agents known to cause vasodilatation and decreased myocardial contractility should be avoided, at least in standard doses.
    The options include:– Propofol and thiopentone may cause cardiovascular instability due to vasodilation and decreased contractility. The use of vasoconstrictor medication may be required, which may result in rebound hypertension and could worsen active bleeding.
    – Midazolam may also cause hypotension, but less marked than propofol and thiopentone and may therefore be an appropriate choice.
    – Fentanyl is a potent analgesic, which does not release histamine and therefore
    maintains cardiac stability. It is hypnotic-sparing and may reduce the
    cardiovascular side-effect profile of induction agents. In addition it obtunds the cardiovascular reflex to laryngoscopy, which is beneficial to prevent an increase in blood pressure and intracranial pressure.
    – Rocuronium is a steroid non-depolarising paralytic agent. Administration does not cause the release of histamine and therefore maintains cardiovascular stability. In addition when used in larger doses of 0.9–1.2mg/kg, the onset of intubating conditions is rapid, occurring between 60 and 90 seconds.
    – Ketamine is a non-competitive N-methyl-D-aspartate (NMDA) receptor
    antagonist at the glutamate pre-synaptic calcium channel and is used to
    provide ‘dissociative anaesthesia’. It has become the drug of choice in the pre-hospital setting in combination with fentanyl due to its cardiovascular effect profile. It causes an increase in systemic vascular resistance and maintains blood pressure via this mechanism. It should be noted that as a calcium antagonist it decreases myocardial contractility and therefore may result in a drop in blood pressure in extreme hypovolaemia. Ketamine has a prolonged length of action of 30–40 minutes negating the need for an infusion. When used in conjunction with controlled mechanical ventilation it does not increase intracranial pressure as described in the 1970s, and may in-fact offer neuro-protection preventing cellular apoptosis.
    Pre-hospital care consensus has advocated the use of fentanyl 3µg/kg, ketamine 2mg/kg and rocuronium 1mg/kg (remembered as 3/2/1). This is considered the safest induction technique at the scene of the accident for the reasons described above. Whether or not we can translate this technique directly to hospital care is not certain, but securing the airway in a poly-trauma patient in hospital has the same priorities as at the scene of the accident. Therefore the most appropriate anaesthesia induction technique is one that has been tried and tested on this group of patients.
17
Q
  1. The obstetric registrar has asked you to review a 22-year-old woman on the postnatal
    ward who underwent a Category 2 Caesarean section for chorioamnionitis 2 days
    ago. She was otherwise previously fit and well. She has a respiratory rate of 28 breaths
    per minute, a heart rate of 100 beats per minute, blood pressure of 92/50mmHg
    and oxygen saturations of 91% on air. She was prescribed intravenous antibiotics
    postoperatively, but had only received one dose before being changed to oral antibiotics as her cannula had tissued and the team had been unable to re-site
    another. Her temperature is 38.7°C and she is complaining of abdominal tenderness.
    What is the next most appropriate line of management?
    A High flow oxygen, blood cultures, intravenous fluids and urgent discussion
    with microbiology consultant
    B High flow oxygen, intravenous fluids and intravenous broad spectrum antibiotics
    C High flow oxygen, intravenous fluids and intravenous paracetamol
    D High flow oxygen, blood cultures, intravenous fluids and oral antibiotics
    E High flow oxygen, blood cultures and intravenous fluids
A
  1. A High flow oxygen, blood cultures, intravenous fluids
    and urgent discussion with microbiology consultant
    With a temperature >38.3°C, a heart rate >90 beats per minute and tachypnoea,
    this lady meets the diagnostic criteria for sepsis based on the general variables as
    outlined by the Surviving Sepsis Campaign. She is also at risk of developing severe
    sepsis, as her blood pressure and oxygen saturations are low. The site of infection
    must be investigated, as she may have developed intra-abdominal sepsis post
    surgery, amongst other possibilities.Genital tract sepsis was the commonest direct cause of maternal death in the last
    triennium, as outlined by the most recent Centre for Maternal and Child Enquiries
    (CMACE) report (2006–2008), with Group A streptococcal disease being the
    responsible pathogen in many cases. Recommendations were made that high dose
    intravenous broad spectrum antibiotics should be administered within 1 hour of
    recognition of sepsis as mortality increases with each hour of delay.
    In this case, the patient has been on antibiotics via an inadequate route. She has
    become more unwell, and blood cultures should be taken and an urgent discussion
    with the consultant microbiologist made to determine the most appropriate
    antibiotics given her recent antibiotic therapy. This treatment should ideally be
    commenced within 1 hour. She is likely to need an escalation in treatment and
    admission to a critical care area may be warranted. Fluid challenges should be given
    and there should be a low threshold for bladder catheterisation to ensure a urine
    output of at least 0.5mL/kg/hour. Oral antibiotic treatment is not appropriate and IV
    paracetamol will not treat the sepsis. Option B is incorrect, as her management must
    include the taking of blood cultures
18
Q
  1. A 4-year-old 18 kg girl is scheduled for elective squint surgery. She was born at 31
    week gestation, was ventilated for 1 week, and then was on CPAP for a month. She
    now suffers from recurrent episodes of wheeze and hospital admissions requiring
    nebuliser therapy.
    She takes salbutamol and beclomethasone inhalers regularly. Her mother reported
    she had just recovered from another viral respiratory tract infection a week ago, but
    no longer had any cough or coryzal symptoms. On examination, she is comfortable
    with no respiratory distress. Her respiratory rate is 16 breaths per minute and her
    oxygen saturation is 98% on air. On auscultation, there is a soft bilateral expiratory
    wheeze.
    The most appropriate management plan for this patient is:
    A Ask the mother to give the patient an extra dose of her salbutamol inhaler
    before induction of anaesthesia
    B Give the patient nebulised salbutamol before induction of anaesthesia
    C Reschedule the surgery for when the patient is 6 weeks from the most recent
    viral respiratory illness
    D Give the patient a dose of intravenous steroid intraoperatively
    E Refer the patient to the paediatric respiratory team for further management
A
  1. C Reschedule the surgery for when the patient is 6 weeks
    from the most recent viral respiratory illness
    Asthma is one of the most common pulmonary disorders encountered by
    paediatric anaesthetists. Asthma patients carry a small but significantly increased
    risk for perioperative complications. Paediatric asthmatic patients require careful
    preoperative evaluation and preparation.
    Essential points to review in the preoperative evaluation are the level of asthma
    control and the current medication regimen. In addition, review of the level of
    activity, use of rescue medications, hospital visits (tracheal intubation or intravenous
    infusions required), allergies, and previous anaesthetic history are important. A Also
    an inquiry regarding cough and sputum production should also occur. Although
    otherwise healthy children can often be anaesthetised safely during an acute
    upper respiratory infection, the risk of bronchospasm in asthmatics is very high.
    They should ideally be postponed 4–6 weeks after such an event, particularly if the
    surgery is non-urgent, as is the case with the patient in this question.
    Preoperative preparation for a controlled asthmatic can include administration of
    inhaled β2 adrenergic agonist 1–2 hours before surgery. For moderately controlled
    asthma, additional optimisation with an inhaled corticosteroid and regular use
    of inhaled β2 agonists 1 week before surgery can be instituted. Poorly controlled
    asthmatics might need addition of systemic corticosteroid 3–5 days before surgery.
19
Q
  1. A 10-year-old 24kg girl is scheduled on your day surgery list for an upper gastro-
    intestinal endoscopy to investigate her unexplained recurrent abdominal pain.
    There is no other significant past medical history. On preassessment, the patient was
    anxious but both mother and patient agreed to a gaseous induction of anaesthesia.
    On arrival in the anaesthetic room, the child is crying, combative and refusing to
    cooperate. After 10 minutes in the anaesthetic room, the child only allowed you to
    put on a pulse oximeter.
    She is the final patient on the morning list, and the endoscopist has a clinic to
    attend in the afternoon.
    Your best plan of action is:
    A Send the child back to the waiting area to have a sedative pre-medication
    B Ask the mother to help restrain the child for a quick gas inductionC Give the child a dose of sublingual midazolam in the anaesthetic room as
    premedication before induction of anaesthesia
    D Give the child a dose of intramuscular ketamine in the anaesthetic room as
    premedication before induction of anaesthesia
    E Reschedule for another day with a plan for midazolam pre-medication on the
    ward
A
  1. E Reschedule for another day with a plan for midazolam
    pre-medication on the ward
    Anaesthetists frequently have to cope with a child who is uncooperative at induction
    of anaesthesia and must be familiar with strategies for preventing and dealing with
    this problem.
    Psychological and pharmacological interventions aimed at reducing preoperative
    anxiety can improve compliance at induction and reduce postoperative behavioural
    changes. Psychological interventions include preoperative ward visit, play therapy,
    parental presence at induction, music, lighting and distraction. Various drugs can
    be used as premedication for the uncooperative child, midazolam being the most
    common. The preferred route of administration is oral, followed by nasal. The rectal
    and intramuscular route should be avoided if possible.
    Uncooperative children are often preschool or young children with an anxious
    temperament, anxious parents, or both. These patients may appear cooperative
    when interviewed in the surgical ward, but then become uncooperative in the
    anaesthetic room or at induction of anaesthesia. Fortunately, they are usually
    amenable to reasoning and encouragement possibly backed up by sedative
    premedication. The use of physical restraint (overpowering), holding still
    (immobilising), and containing (preventing escape or self-harm) in children raises
    ethical, legal, and practical problems, and should only be used as a last resort.
    If the surgery is elective, as in the case above, then the option of postponing the
    procedure should be considered. Postponing the procedure gives more time for
    planning, but may not be convenient for the parents. Giving premedication in a day
    surgery environment may not be appropriate, so rescheduling the operation for
    another day, as inpatient, is the best plan of action in this case.
20
Q
  1. A 6-year-old 20 kg girl is scheduled to have an emergency laparoscopic
    appendicectomy. She is clinically stable and appears comfortable at rest. You
    discover in your preoperative assessment that the patient’s maternal uncle has a
    possible history of malignant hyperthermia, but the patient and both her parents
    have not been investigated for malignant hyperthermia susceptibility (MHS).
    The most appropriate anaesthetic management is:
    A Postpone anaesthesia and surgery until the possibility of MHS in the patient
    has been investigated
    B Postpone anaesthesia and surgery until more information is available about
    the uncle’s history of malignant hyperthermia
    C Proceed with anaesthesia and surgery, but with modified anaesthesia
    technique to avoid known triggers for malignant hyperthermia
    D Proceed with anaesthesia and surgery. Malignant hyperthermia is not
    maternally inherited, so modification of anaesthetic technique is not required.
    E Proceed with anaesthesia and surgery, but with a high vigilance for malignant
    hyperthermia.
A
  1. C Proceed with anaesthesia and surgery, but with
    modified anaesthesia technique to avoid known triggers
    for malignant hyperthermia
    Malignant hyperthermia (MH) is an inherited disorder of skeletal muscle that can
    be pharmacologically triggered to produce a potentially fatal combination of
    hypermetabolism, muscle rigidity and muscle breakdown. Malignant hyperthermia
    susceptibility (MHS) is inherited in an autosomal dominant fashion. However, a
    parent with MHS may not necessarily have a positive history of MH. Anaesthetic
    technique must be modified to avoid known triggers for MH (halothane, enflurane,
    isoflurane, sevoflurane, desflurane, and succinylcholine) in any cases of suspected
    or confirmed MHS. The anaesthetic machine should be prepared by removal of
    vapourisers and flushing through the machine and ventilator with 100% oxygen at
    maximal flows for 20–30 minutes, and a new breathing circuit should be used.
    The key to successful management of MH is its early diagnosis and the rapid
    instigation of several modes of treatment simultaneously. Administration of volatile
    anaesthetics should be discontinued and the patient’s lungs hyperventilated using 100% oxygen with fresh gas flows and type of breathing circuit optimised to
    eliminate the anaesthetic from the body. Anaesthesia should be maintained with
    intravenous drugs while surgery is concluded as rapidly as possible. Active cooling
    measures should be commenced. At the onset of treatment, one member of staff
    must be assigned to the preparation of dantrolene sodium for infusion. Repeated
    doses of dantrolene (1mg/kg up to maximum of 20mg) should be administered
    intravenously as soon as possible until the tachycardia, rise in CO2 production and
    pyrexia start to subside. Up to 10mg/kg may be required.
    Postponing surgery for further information and investigation is not an option in this
    case due to the urgency of the surgery. Proceeding with an ‘MH-safe’ anaesthetic is
    the most appropriate approach in this clinical scenario.
21
Q
  1. A 73-year-old woman suffering with depression and poorly controlled chronic
    back pain who is taking paracetamol, diclofenac and fluoxetine is started on
    tramadol. The following day, she presents to the emergency department with
    tremor, confusion and restlessness. On examination she is febrile, hyperreflexic
    and has mydriasis.
    What is the most likely cause of her symptoms?
    A Opioid toxicity
    B Opioid withdrawal
    C Hyponatraemia
    D Serotonin syndrome
    E Anaphylaxis
A
  1. D Serotonin syndrome
    Serotonin syndrome is a potentially lethal condition resulting from excess agonist
    activity at central and peripheral serotonergic receptors. It can result from therapeutic
    drug use, intentional self-poisoning or interactions between drugs, many of which
    anaesthetists are involved with. The syndrome is characterised by neuromuscular
    excitability, autonomic hyperactivity, and altered mental status. It is a clinical
    diagnosis and the presence of tremor, clonus, or akathisia without extrapyramidal
    signs should lead clinicians to consider the syndrome, particularly if the patient is
    taking drugs known to elevate serotonin levels. Tramadol is a commonly prescribed
    analgesic and works by activation of central μ-opioid receptors. In addition to its
    opioid receptor effects, tramadol also inhibits neuronal reuptake of serotonin and
    noradrenaline. This property of tramadol can lead to elevated plasma serotonin
    levels which increases the risk of developing the serotonin syndrome. When
    tramadol is taken in conjunction with serotonergic agents like SSRIs (such as in
    the case above), this risk increases. Treatment is generally supportive and involves
    removing the offending agent(s) and controlling the agitation, autonomic instability
    and hyperthermia. The antihistamine cyproheptadine, which is also a serotonin
    antagonist, is reserved for severe cases. Serotonin syndrome is the most likely
    diagnosis in the case above since there are clinical signs of neuromuscular excitability,
    autonomic hyperactivity and altered mental status in a patient known to be taking
    two agents which can increase serotonin levels.
    Opioid toxicity can occur with tramadol administration, although symptoms such
    as drowsiness and lethargy would be more likely than the restlessness described.
    Furthermore, miosis as opposed to mydriasis would be expected on examination
    of the pupils. Hyperreflexia, tremor and pyrexia are also not typical presentations of
    opioid toxicity.
    Opioid withdrawal shares many of the symptoms and signs seen in the serotonin
    syndrome such as restlessness, tremor, mydriasis and tachycardia. The temporal
    relationship between tramadol use and the symptoms in the above scenario
    however makes opioid withdrawal unlikely. Withdrawal symptoms usually occur
    following cessation of opioids after several weeks of steady use in which physical
    dependence is attained. In the above scenario tramadol was only taken for 1 day,
    and there was no history of its cessationThe symptoms associated with hyponatraemia are predominantly neurological
    due to cerebral oedema associated with a reduced serum osmolality. The muscle
    hyperactivity and confusion as seen in the above case are typical symptoms of
    hyponatraemia which can progress to seizures, coma and respiratory arrest if
    the hyponatraemia is not corrected. Tramadol can cause hyponatraemia on rare
    occasions, thought to be due to opioid and serotonin receptor induced ADH release.
    The fever described in the above case however is not typical of hyponatraemia, but
    is frequently seen in the serotonin syndrome due to autonomic hyperactivity.
    Anaphylaxis should always be in the differential diagnosis for any patient who
    becomes unwell after starting a new medication. In the above case however, the
    tremor and hyperreflexia suggest an alternative diagnosis.
22
Q
  1. A 41-year-old woman presents for repeated wide local excision for breast cancer,
    and is due to have adjuvant radiotherapy. Her past medical history includes
    diabetes and depression. She is worried about the operation, especially pain after
    her surgery.
    Which of the following is not a risk factor for chronic post-surgical pain?
    A Diabetes
    B Fear of surgery
    C Repeated surgery
    D Younger age
    E Adjuvant radiotherapy
A
  1. A Diabetes
    Chronic post-surgical pain (CPSP) is recognised as:
    • Pain developing after a surgical procedure
    • Pain of at least 2 months duration
    • Other causes of pain excluded (such as infection)
    • Pain continuing from a pre-existing pain problem excluded
    ○ Risk factors for development of CPSP can be patient factors or surgical factors.
    ° Surgical factors include type of procedure (breast surgery, amputation,
    thoracotomy), length of surgery and repeat surgery for the same pathology. °Surgical approach is also important, as the use of a laparoscopic technique results in less CPSP for cholecystectomy and hernia repairs. The use of adjuvant radiotherapy is also associated with a significantly increased risk of CPSP.
    Patient factors include age (CPSP after breast cancer surgery decreases by 5% for
    each yearly increase in the patient’s age ), genetic susceptibility and psychosocial
    risk factors. For example, fear of surgery after breast surgery is associated with
    worse pain and a higher risk of progression to CPSP. Additionally, the severity of
    postoperative pain positively correlates with the incidence of development of CPSP.
    Diabetes is not a recognised risk factor for the development of CPSP.
23
Q
  1. A 65-year-old man presents to the pain clinic with long-standing poorly controlled
    lower back pain. He is frightened by the painful sensations and admits to feeling
    depressed since he is no longer able to walk unaided.
    Which of the following is the most appropriate assessment tool to evaluate his
    painful experience?
    A Numeric rating scale
    B Visual analog scale
    C Verbal descriptor scale
    D McGill pain questionnaire
    E Wong–Baker FACES scale
A
  1. D McGill pain questionnaire
    Pain is a complex, subjective experience which often requires specialised assessment
    tools to fully evaluate and quantify. Numerous pain rating scales have been developed
    over the years and it is important as anaesthetists to appreciate the context in which
    they should be used. Unidimensional pain scales are useful for evaluating acute pain
    of clear aetiology (e.g. postoperative pain) since they allow quick assessment of pain
    intensity and response to treatment. However, they are less effective in evaluating
    chronic pain, since they often fail to measure the associated affective and disablingcomponents. Multidimensional pain scales are more appropriate in these cases since
    they allow measurement of these other facets of the pain experience.
    The McGill pain questionnaire is one of the most extensively tested
    multidimensional scales, and is the most appropriate tool to use in the above clinical
    scenario. The three-part questionnaire assesses not only the sensory aspects but
    also the affective component of pain which the above patient is suffering from.
    This assessment tool may also help identify whether there are any specific pain
    syndromes (such as neuropathic pain) present.
    The numeric rating scale is a commonly used unidimensional pain scale where
    patients rate their pain intensity on a ten point scale with a score of ten representing
    ‘the worst imaginable pain’. It is easy to use and has been validated in numerous
    settings and pain types. However, it only measures one dimension of pain which
    limits its usefulness in the chronic pain setting. It is also less reliable in patients with
    cognitive impairment.
    The visual analog scale is an assessment tool composed of a ten centimetre line
    representing the spectrum of pain intensity from ‘no pain’ to the ‘worst pain
    imaginable’. Patients are instructed to mark a point on the line which corresponds
    to their level of pain, and the distance between this mark and zero is measured. This
    tool is sensitive for variations in pain intensity with treatment and is reproducible.
    However, like the numeric rating scale, it only measures one component of the
    complex multidimensional nature of chronic pain.
    The verbal descriptor scale is a six point categorical scale of descriptive words from
    ‘no pain’ to ‘worst possible pain’ which the patient can use to express their pain
    experience. It allows for a rapid assessment of pain intensity and is easy to use at the
    bedside. However, it forces the patient to use someone else’s words to describe their
    pain and does not measure the multidimensional components of chronic pain.
    The Wong–Baker FACES scale is another categorical scale with faces conveying
    expressions of pain with increasing severity. Patients are instructed to select the face
    which best matches how they are feeling to provide a crude measure of their pain
    experience. It is a useful assessment tool for children and patients with cognitive
    impairment who may otherwise have difficulties in quantifying their pain but it is
    not the most appropriate scale to use in patients with chronic pain.
24
Q
  1. You are asked to review an 84-year-old woman overnight on the ward with a right
    hip fracture. The orthopaedic core trainee is unable to control her pain despite
    administering paracetamol and 15mg Oramorph.
    What is the most appropriate next step for managing this patient’s pain?
    A Add gabapentin
    B Give a stat one off dose of ibuprofen
    C Start a patient controlled analgesia (PCA)
    D This patient needs emergency surgery
    E Perform a nerve block
A
  1. Perform a nerve block
    ○ Preoperative pain management for fractured neck of femur patients is a significant problem. As such, the National Institute for Health and Care Excellence (NICE) have issued guidelines (CG124) aiming to optimise analgesic management for these patients whilst awaiting definitive surgery. This guideline states:
    • Offer immediate analgesia to all patients presenting to hospital with a suspected hip fracture, including people with cognitive impairment
    • Ensure analgesia is sufficient to allow movements necessary for investigations and nursing caret Offer paracetamol 6 hourly unless contraindicated
    • Offer additional opiates if paracetamol alone does not provide sufficient
    preoperative pain relief
    • Considering adding a nerve block if paracetamol and opioids do not provide sufficient preoperative pain relief, or to limit opioid dosage
    • Non-steroidal anti-inflammatory drugs (NSAIDs) are not recommended
    ○ This patient would therefore most likely benefit from an ultrasound-guided femoral nerve block or a fascia iliaca block. This is relatively simple to perform and has been demonstrated to have a significant impact on preoperative analgesia.
25
Q
  1. A 59-year-old man with a 2 year history of type I complex regional pain syndrome affecting his left leg presents to the pain clinic after a failed trial of epidural injections and physiotherapy. He has a fentanyl patch and is taking paracetamol, ibuprofen, amitriptyline and gabapentin. Despite this, he suffers from severe debilitating leg pain, allodynia and hyperalgesia.
    What intervention is the most appropriate next step in managing his symptoms?
    A Spinal cord stimulation
    B Radiofrequency lumbar sympathectomy
    C Below knee amputation
    D Guanethidine-sympathetic blockade
    E Non-invasive brain stimulation
  2. You are pre-assessing a 6-year-old child
A
  1. A Spinal cord stimulation
    ○ Complex regional pain syndrome (CRPS) is a debilitating, painful condition which is classified into type I and II subtypes, depending on the absence or presence of an antecedent peripheral nerve injury respectively. The lead symptom of CRPS is limb-confined pain, but the syndrome also encompasses autonomic, motor, skin and bone changes. If the pain is unrelenting and the physical impairment persists for more than 2 years, the condition is considered long-term.
    ○ The aim of medication is to minimise pain and support physical rehabilitation.
    ○ Although no drugs are licensed to treat CRPS in the UK, national guidelines encourage the use of drugs targeting neuropathic pain if simple medication is unsuccessful after 4 weeks.
    ○ In the UK, the only National Institute for Health and Care Excellence (NICE) approved method to treat CRPS is spinal cord stimulation, and should be considered in patients who have not responded to appropriate integrated management.
    ○ Stimulation of the spinal cord is achieved by application of an electrical current to the dorsal columns of the spinal cord through a catheter inserted into the epidural space.
    °The exact mechanism of action is unclear but some investigators suggest that spinal cord stimulation may activate Aβ afferents which modulate the transmission of pain based on the gate control theory.
    °Others suggest that spinal cord stimulation may block
    spinothalamic tract transmission or enhance descending inhibitory mechanisms.
    ○ In the above case where simple analgesia and medication targeting neuropathic pain have failed, spinal cord stimulation is an appropriate next management step. In order to assess the potential benefit of spinal cord stimulation, the electrodes are initially stimulated by an external stimulating device prior to permanent pulse generator insertion. Patient satisfaction is generally high, although there is some evidence that the efficacy of this treatment generally declines over time.
    ○ The autonomic vasomotor changes seen in CRPS have led observers to previously view the associated pain as sympathetically mediated, and although common in early CRPS, it is actually rare in long-term cases.
    °The use of guanethidine (which depletes the limb autonomic nerve endings of noradrenaline) to achieve chemical sympathectomy in the affected limb has been shown to be ineffective in randomised controlled trials, and is not recommended
    ° Lower limb sympathectomy can also be achieved by creating a thermal lesion from the application of a high frequency current to lumbar sympathetic ganglia via percutaneous electrodes.
    ° Proponents of this intervention (termed radiofrequency ablation), suggest that it is less invasive than surgical resection and can help break
    the cycle of pain.
    ° The most recent Cochrane review however, judged that there was
    no evidence from controlled trials that sympathectomy (including radiofrequency) was no more effective than placebo or no treatment.
    Amputation should not be used to provide pain relief in CRPS, and should only be considered in rare cases of intractable infection of the affected limb. Amputation may worsen CRPS, with symptoms recurring in the stump.
    An interesting approach to tackle the symptoms of CRPS is through the use of non-invasive brain stimulation. The treatment is based on the hypothesis that CRPS is primarily a disease of cortical organisation which results in changes in the way somatosensory systems process tactile, noxious and thermal information. The aim of non-invasive brain stimulation is to induce cortical modulation through the application of a repetitive current to improve symptoms. This treatment is an interesting concept, however further research is needed to evaluate its efficacy.
26
Q
  1. You are pre-assessing a 6-year-old child in the day unit for re-do strabismus
    surgery to the right eye. The mother tells you that the child underwent the
    procedure six months prior and had to stay overnight due to intractable post-
    operative nausea and vomiting.
    Which of the following is least likely to prevent a repeat of this?
    1A Avoidance of perioperative opioids
    B Atropine 20µg/kg at induction
    C Ondansetron 0.15mg/kg
    D Sevoflurane maintenance over desflurane
    E Ondansetron 0.15mg/kg plus dexamethasone 0.15mg/kg
A
  1. D Sevoflurane maintenance over desflurane
    Strabismus surgery is one of the most common paediatric ophthalmic operations.
    The oculocardiac reflex and postoperative nausea and vomiting (PONV) are major
    anaesthetic concerns in this procedure.
    The oculocardiac reflex occurs due to traction on the extraocular muscles, which
    causes bradycardia. This can be attenuated by release of surgical traction or the
    administration of anticholinergic medication such as atropine. Hypercarbia can also
    increase the incidence of bradycardia, therefore ventilating the patient to maintain a
    normal CO2 is a technique often used.
    PONV is more likely to occur in children who demonstrate the oculocardiac
    reflex, therefore preventing this reflex should prevent PONV. Atropine 20µg/kg
    is advocated as a result, and so option B is not the correct answer. Intraoperative
    intravenous fluids, avoiding opioid analgesia and using antiemetic prophylaxis helps
    to reduce PONV incidence. Evidence suggests that ondansetron in combination with
    dexamethasone is more effective than ondansetron alone.
    All the volatile agents increase the risk of PONV and there is currently no evidence to
    show that any one is less emetogenic than the other, hence D is the correct answer.
27
Q
  1. You are called to the emergency department to assess a 65-year-old non-
    insulin dependent diabetic man presenting with an ischaemic foot. He also has
    hypertension and exertional angina. He admits to getting progressively short of
    breath over the last 6 months but he is able to climb one flight of stairs without
    stopping. Physical examination reveals no basal crackles and heart sounds are
    normal.
    What is the next most appropriate step in the management of his acutely ischaemic
    foot?
    A Delay surgery until an echocardiogram can be performed
    B Proceed to surgery without delay
    C Proceed to surgery after discussing the case with your consultant
    D Proceed to surgery after booking a bed in HDU/ITU
    E Delay surgery until an arteriogram to identify the location of the blockage can
    be performed
A
  1. C Proceed to surgery after discussing the case with your
    consultant
    The patient has two risk factors for moderate postoperative cardiac risk: stable
    congestive heart failure and stable angina. According to the ACC/AHA 2007
    guidelines for managing cardiac risk of patients for non-cardiac surgery, surgical
    risk and urgency are used in conjunction with clinical risk and clinical assessment of
    exercise tolerance to outline the best approach of managing complex situations such
    as this. The patients with the highest risk of major adverse cardiac events (MACE), i.e.
    death or myocardial infarction, are those that possess several clinical diagnoses:
    t Unstable angina
    t Overt congestive cardiac failure
    t Uncontrolled arrhythmias
    t Severe stenotic valvular disease
    t Recent myocardial infarction (within 4 weeks).
    This only applies to non-emergent situations. An emergency would override those
    considerations in view of the risk of MACE being higher if the operation would be
    delayed.
    Stable heart failure, stable angina, rate controlled atrial fibrillation, chronic renal
    impairment or history of cerebrovascular event are risk factors for MACE and
    increase the burden of post-operative morbidity, however delaying life or limb
    saving operations to further investigate them would expose the patient to a higher
    than necessary risk of MACE. The evidence presented in the guidelines suggests
    that an initial assessment and improvement of the above conditions may result in a
    decrease of MACE.
    In this case it is a potential life or limb saving procedure has a lower risk than waiting
    for a cardiology opinion to optimise his heart failure and angina. Conditioning life
    or limb saving surgery to availability of a high care bed is not advisable. Early senior
    involvement is the most likely step towards a safe and effective intra-operative
    management for this patient.
28
Q
  1. An obese 45-year old patient has undergone an inguinal hernia repair under
    general anaesthesia and a first generation supraglottic airway device was used. In
    recovery, he becomes hypoxic and short of breath. You are suspecting a pulmonary
    aspiration of gastric contents.
    Which of the following lung segments is most likely to be contaminated following
    an episode of aspiration during a general anaesthetic?
    A Apical segments of the lower lobes
    B Posterior basal segments of the lower lobes
    C Lateral segment of the right middle lobe
    D Lateral basal segments of the lower lobes
    E Apical segments of the upper lobes
A
  1. A Apical segments of the lower lobes
    Pulmonary aspiration of gastric contents is one of the most serious complications
    after general anaesthesia. Acid aspiration may cause immediate lung tissue injury
    and subsequent severe inflammatory response.
    The anatomy of the lung lobes and bronchopulmonary tree affects zonal
    contamination if aspiration of gastric contents happens.
    The trachea is a 10–12cm long tube that connects the larynx to the lungs. In the first year
    of life, the tracheal diameter is 3mm or less. The diameter then increases by about 1mm
    per year until it reaches the adult size (around 20–25mm). The trachea comprises fifteen
    to twenty C-shaped cartilaginous rings. These rings are incomplete posteriorly allowing
    the trachea to collapse slightly during the passage of the food in the esophagus. It commences at the cricoid cartilage, level with the 6th cervical vertebra (C6), and
    divides into right and left main bronchi at the level of the 5th thoracic vertebra (T5).
    The right main bronchus (RMB) is about 3cm long. It is shorter, wider and aligned
    more vertically than the left main bronchus. Therefore, tracheal intubation and
    foreign body inhalation are more likely to happen in the right main bronchus instead
    of the left. The RMB gives off to 10 bronchopulmonary segments (3 in the upper lobe,
    2 in the middle lobe and 5 in the lower lobe). After around 2.5–3cm, the RMB gives off
    the right upper lobe bronchus (RULB). The RULB is further divided into apical, anterior
    and posterior segments after. Because the RULB arises early from the right main
    bronchus, it is most at risk from occlusion by a right-sided double lumen tube.
    The RMB then gives off the right middle lobe bronchus (RMLB). The RMLB is oriented
    forwards and downwards and further divides into medial and lateral segments. The
    RMB then continues on it’s downwards course as the right lower lobe bronchus
    which gives off five segments (apical, medial basal, anterior basal, lateral basal and
    posterior basal).
    The left main bronchus (LMB) is around 5cm in length, and the anatomy is slightly
    different from the right lung. After 5cm, the LMB gives off the left upper lobe
    bronchus, which bifurcates into a superior division and a lingular division. The
    superior division gives off the apical, posterior and anterior segments of the upper
    lobe, while the lingular division gives off the superior and the inferior segments. The
    left lower lobe bronchus (LLLB) differs from the right lower lobe bronchus in that it
    gives four segments instead of five (apical, anterior basal, lateral basal and posterior
    basal). The medial basal segment is usually small and arises with the anterior
    segments. Technically, this means there are four rather than five bronchopulmonary
    segments on the left (see Figure 6.3). Zonal contamination of the lung lobes and the bronchopulmonary segments after
    aspirating is dependent on the patient’s position during the aspiration.
    In a supine patient the apical segment of the lower lobe is more likely to be
    contaminated because of the direct posterior orientation of the segment. If the
    patient is prone, then aspiration is more likely to affect the right middle lobe or the
    lingula because of their forward and downward projection. If in the upright sitting
    position, the lateral or posterior basal segments of the lower lobes will be the site of
    the problem, and in the lateral position, the upper lobes would be contaminated.
29
Q
  1. A 17-year-old girl with a body mass index (BMI) of 15 has been brought to the
    emergency department with a heart rate of 42 beats per minute and a blood
    pressure of 76/34mmHg. Her respiratory rate is 10 breaths per minute and she is
    complaining of epigastric discomfort. Her investigations reveal an atrioventricular
    block and her blood gases demonstrate a metabolic alkalosis. Her mother states
    that she has lost weight and has had amenorrhoea for the past six months.
    The most likely diagnosis is:
    A Ectopic pregnancy
    B Duodenal perforation
    C Anorexia nervous
    D Acute hypothyroidism
    E Opioid overdose
A
  1. C Anorexia nervosa
    ○ Anorexia nervosa is a psychiatric disorder which affects numerous systems in the body and involves the following diagnostic criteria:
    • Body weight is less than 15% of expected or body mass index (BMI) less than or equal to 17.5
    • Self-induced weight loss involving food avoidance, purging, self-induced
    vomiting or using diuretics
    • Distorted body image
    • Multiple endocrine dysfunctions involving hypothalamic-pituitary-gonadal axis
    • Delayed pubertal events if it manifests prior to puberty

○ The condition affects numerous physiological systems in the body including:
• Cardiovascular – hypotension, bradycardia, myocardial dysfunction, mitral valve prolapse, cardiomyopathy and arrhythmias
• Respiratory – metabolic alkalosis, decreased lung compliance, aspiration
pneumonia
• Gastrointestinal – enlarged salivary glands, dental caries, Mallory–Weiss tears, oesophagitis, gastric dilatation/perforation, increased amylase, abnormal liver function tests
• Renal – proteinuria, reduced serum Na+, K+, Cl-, H+, Mg2+, Ca2+ and renal calculi
• Endocrine – reduced FSH, LH, GnRH, T3, T4 and glucose, increased serum cortisol
• Haematological – anaemia, leucopenia, thrombocytopenia
• Neurological – reduced cognitive function, seizures, coma, neuropathy
• Musculoskeletal – osteopenia, pathological fractures, myalgia
○ An ectopic pregnancy, duodenal perforation or peritonitis would lead to tachycardia along with hypotension.
○ Hypothyroidism would lead to a reduced basal metabolism,
which would prevent weight loss. An opioid overdose would cause respiratory acidosis as the major component in an arterial blood gas

30
Q
  1. A 33-year-old man who suffers from chronic alcohol and drug abuse was admitted to the intensive care unit with a head injury 5 days ago. He has been intubated and ventilated since admission and has been receiving enteral nutrition. Whilst on the unit he has developed refeeding syndrome.
    Which of the following is the most appropriate management in this patient?
    A Omitting fluids containing potassium
    B Avoiding thiamine in his enteral feed
    C Infusion of 10% dextrose to correct hypoglycaemia
    D Infusion of phosphate
    E Infusion of calcium chloride to counter hyperkalaemia
A
  1. D Infusion of phosphate
    ○ Refeeding syndrome is defined as the shift of electrolytes and fluids that can occur in patients who are malnourished and have been started on artificial feeding (enteral or parenteral) and could have potentially fatal consequences.
    ○ The hallmark feature is hypophosphatemia but may also include hypokalaemia, hypomagnesaemia along with abnormal sodium and fluid balance.
    ○ Patients with long-standing nutritional deficiencies like anorexia, chronic alcoholics, oncological disorders and chronic malnutrition are at high risk.
    ° Malnourishment increases ketone bodies as fatty acids are used as the primary source of energy.
    ° Intracellular minerals such as phosphate are depleted, although the serum levels may be normal due to contraction of the intracellular compartment and reduced excretion.
    ° On refeeding, glycaemia causes insulin secretion which stimulates glycogen, fat and protein synthesis.
    ° This leads to consumption of minerals like phosphate and magnesium, as well as co-factors such as thiamine.
    ° Insulin secretion causes intracellular migration of glucose and water along with a phosphate and magnesium shift thereby causing acute depletion.
    ○ Management of refeeding syndrome consists of sequential screening of serum levels of potassium, magnesium, calcium and potassium. The levels of phosphate, potassium, magnesium and calcium need to be replenished along with administration of thiamine.
    ○ Excessive administration of glucose in a patient suffering from refeeding syndrome could lead to hyperglycaemia induced diuresis, dehydration, metabolic acidosis and ketoacidosis.
    ○ In this patient, it is therefore most appropriate to supplement the probable hypophosphatemia with a phosphate infusion.